Medical-Surgical Nursing Exam 4

Practice Mode

Welcome to your Medical-Surgical Nursing Exam 4! This exam is carefully curated to help you consolidate your knowledge and gain deeper understanding on the topic.

 

Exam Details

  • Number of Questions: 50 items
  • Mode: Practice Mode

Exam Instructions

  1. Practice Mode: This mode aims to facilitate effective learning and review.
  2. Instant Feedback: After each question, the correct answer along with an explanation will be revealed. This is to help you understand the reasoning behind the correct answer, helping to reinforce your learning.
  3. Time Limit: There is no time limit for this exam. Take your time to understand each question and the corresponding choices.

Tips For Success

  • Read each question carefully. Take your time and don't rush.
  • Understand the rationale behind each answer. This will not only help you during this exam, but also assist in reinforcing your learning.
  • Don't be discouraged by incorrect answers. Use them as an opportunity to learn and improve.
  • Take breaks if you need them. It's not a race, and your understanding is what's most important.
  • Keep a positive attitude and believe in your ability to succeed.

Remember, this exam is not just a test of your knowledge, but also an opportunity to enhance your understanding and skills. Enjoy the learning journey!

 

Click 'Start Exam' when you're ready to begin. Best of luck!

💡 Hint

When a client experiences vertigo, the priority is to prevent injury by ensuring their safety and stability. This can be achieved by helping them to sit or lie down in a secure position.

1 / 50

1. When a client experiences vertigo while walking down the hall, what is the priority nursing action?

💡 Hint

Exophthalmos is a condition that causes the eyes to bulge outward. Consider what change in the tissues around the eyes could lead to this protrusion.

2 / 50

2. What is the underlying cause of exophthalmos in a patient with thyroid disease?

💡 Hint

In case of a possible transfusion reaction, it is crucial to quickly stop the transfusion and take measures to prevent further complications while maintaining the client's IV access.

3 / 50

3. While caring for a client receiving a blood transfusion, the transfusion was initiated 30 minutes ago at a rate of 100 ml/hr. The client starts complaining of lower back pain, headache, and increased restlessness. What should be the nurse's first course of action?

💡 Hint

Conjunctivitis is highly contagious, and proper hygiene plays a vital role in preventing its spread. It is crucial to avoid sharing personal items that come into contact with the eyes or face.

4 / 50

4. Which statement demonstrates that a child diagnosed with conjunctivitis has comprehended the nurse's instructions?

💡 Hint

Consider the typical side effects of long-term steroid therapy, particularly those related to body fat distribution.

5 / 50

5. What assessment finding should a nurse expect in a patient who has been receiving long-term steroid therapy?

💡 Hint

Focus on the precautions necessary to minimize the risk of infection in a child with a weakened immune system due to chemotherapy.

6 / 50

6. When discharging a child with leukemia who has just started chemotherapy, what guidance should the nurse include in the teaching plan for the child's parents?

💡 Hint

After applying anesthesia to the eye, it is essential to avoid actions that could cause harm or discomfort, particularly during the initial period of reduced sensation.

7 / 50

7. Upon anesthetizing a client's eye, which crucial guidelines should the nurse provide to the client?

💡 Hint

In the case of DIC, consider the type of fluid that may help address coagulation imbalances and replenish clotting factors.

8 / 50

8. For a client in the ICU diagnosed with disseminated intravascular coagulation (DIC), the nurse should anticipate administering which fluid?

💡 Hint

Focus on understanding the main manifestations of Meniere's disease and how their frequency and severity can impact the patient's well-being.

9 / 50

9. When admitting a patient with Meniere's disease, which assessment is of the utmost importance for the nurse to conduct?

💡 Hint

Consider the effects of different eye drops on a patient's ICP and identify the one with the potential to exacerbate the condition.

10 / 50

10. For a patient with elevated intracranial pressure (ICP), the nurse should be cautious about administering which type of eye drop?

💡 Hint

When suctioning through an endotracheal tube, the main objective is to clear secretions while minimizing the risk of airway injury.

11 / 50

11. Which statement accurately characterizes the process of suctioning through an endotracheal tube?

💡 Hint

Consider the primary symptoms of a thyroid crisis, which include elevated body temperature, and think about interventions that may exacerbate this symptom.

12 / 50

12. While caring for a patient experiencing a thyroid crisis, which order should the nurse question?

💡 Hint

Helping a COPD patient manage daily activities and energy levels is an essential aspect of home care.

13 / 50

13. The spouse of a patient with COPD is concerned about providing care at home. Which statement by the nurse offers the most accurate information?

💡 Hint

Metformin's primary function is to help control blood sugar levels. Taking it with meals can help reduce potential side effects related to digestion and absorption of carbohydrates.

14 / 50

14. The nurse should educate the client about metformin (Glucophage), emphasizing that it:

💡 Hint

Focus on the primary symptom of diabetes insipidus.

15 / 50

15. What is the priority nursing diagnosis for a patient admitted to the hospital with a diagnosis of diabetes insipidus?

💡 Hint

Keep in mind the potential side effects of levothyroxine and the importance of addressing any signs of adverse reactions promptly.

16 / 50

16. What is the most important instruction for a patient prescribed daily levothyroxine (Synthroid) regarding the administration of the drug?

💡 Hint

Pheochromocytoma is a tumor of the adrenal gland that causes excessive secretion of catecholamines, leading to elevated blood pressure. After adrenalectomy, blood pressure fluctuations may occur as the body adjusts to the removal of the tumor.

17 / 50

17. Following an adrenalectomy for a client diagnosed with pheochromocytoma, the nurse should meticulously evaluate this patient for:

💡 Hint

Consider the possible causes of these symptoms and the importance of evaluating the patient's nutritional intake to identify potential deficiencies or imbalances.

18 / 50

18. When assessing a 25-year-old woman presenting with dizziness, weakness, and palpitations, what is the initial priority for the nurse to evaluate while obtaining her health history?

💡 Hint

Consider the changes in blood gas measurements that might suggest a patient is having difficulty clearing their secretions, which could impact gas exchange.

19 / 50

19. On the first postoperative day following a right lower lobe (RLL) lobectomy, the client breathes and coughs but struggles to bring up mucus. What indicates that the client is not effectively clearing secretions?

💡 Hint

In the event of a chemical exposure to the eyes, rapid action is necessary to minimize damage. Consider the most effective way to reduce the impact of the chemical on the eye.

20 / 50

20. A client contacts the nurse after accidentally splashing an unknown chemical into their eyes. What is the most crucial action the nurse should advise the client to take immediately?

💡 Hint

When infusing blood products, it is essential to select a compatible IV fluid that will not cause hemolysis or other complications.

21 / 50

21. Before administering a unit of packed red blood cells, the nurse is preparing to initiate an IV infusion. Which fluid should the nurse choose to maintain the infusion prior to hanging the blood unit?

💡 Hint

Consider the factors that can exacerbate angle-closure glaucoma, including the potential side effects of certain medications.

22 / 50

22. What is an important aspect to include in the nursing care of a patient with angle-closure glaucoma?

💡 Hint

Consider the main issues faced by patients in a sickle cell crisis and focus on addressing the complications related to circulation, oxygen supply, and discomfort.

23 / 50

23. A patient experiencing a sickle cell crisis is admitted to the emergency department. What should be the priorities of care for this client?

💡 Hint

Consider the purpose of a myringotomy and its role in addressing issues related to middle ear infections and fluid accumulation.

24 / 50

24. What is the primary goal of a myringotomy procedure that the nurse should discuss with the parents of a child scheduled for the surgery?

💡 Hint

Consider the common physical signs that may indicate low blood sugar levels in a patient, such as perspiration.

25 / 50

25. How can a nurse determine if a patient receiving 20 units of NPH insulin subcutaneously at 7:00 AM daily is experiencing a hypoglycemic reaction at 3:00 PM?

💡 Hint

This question pertains to the impact of prednisone, a corticosteroid, on a diabetic patient. Think about how corticosteroids affect glucose metabolism and the potential implications for someone with diabetes.

26 / 50

26. A patient diagnosed with type 2 diabetes has been prescribed prednisone to alleviate severe arthritis pain. What is a potential change that necessitates careful monitoring by the nurse?

💡 Hint

Tetany can occur following a thyroidectomy due to a drop in calcium levels. Consider which medication can effectively address this calcium deficiency.

27 / 50

27. What medication should the nurse have on hand for the emergency treatment of tetany in a patient who has undergone a thyroidectomy?

💡 Hint

Consider the potential risks and complications that may arise after ear surgery, and focus on the primary safety concerns associated with a patient's mobility and stability.

28 / 50

28. In order to ensure the safety and well-being of a patient following a stapedectomy, which aspect should be incorporated into the nursing care plan?

💡 Hint

Consider the risk factors for iron deficiency anemia and focus on the nutritional aspect, especially in early childhood.

29 / 50

29. Which individual has the highest probability of developing iron deficiency anemia?

💡 Hint

Consider the location of the thyroid gland and the potential complications that may arise after surgery. Focus on monitoring the surgical site and addressing any issues that could arise postoperatively.

30 / 50

30. What measures should be taken in postoperative nursing care following a thyroidectomy?

💡 Hint

Consider how the body utilizes carbohydrates during exercise and the impact this has on insulin requirements for diabetic patients.

31 / 50

31. What should the nurse instruct a diabetic patient about the role of exercise in their treatment program?

💡 Hint

SIADH is characterized by excessive release of antidiuretic hormone, which affects sodium balance and osmolality in the body.

32 / 50

32. What laboratory values would a nurse expect to find when assessing a patient with Syndrome of Inappropriate Antidiuretic Hormone (SIADH)?

💡 Hint

Hearing aids primarily work by amplifying sounds to make them more accessible for individuals with hearing loss. However, they do not necessarily improve the actual ability to hear or comprehend sounds.

33 / 50

33. When educating a client and their family about using a hearing aid, on which key piece of information will the nurse base the teaching?

💡 Hint

Meniere's disease is characterized by symptoms like vertigo, hearing loss, and tinnitus. To help manage these symptoms, a diuretic medication is often prescribed to reduce fluid buildup in the inner ear.

34 / 50

34. Which medication should a nurse expect to administer to a client diagnosed with Meniere's disease?

💡 Hint

The HbA1c test measures the average blood sugar levels over the past three months. Consider the preparation needed for this test compared to tests that require fasting or multiple blood draws.

35 / 50

35. What essential information should the nurse provide to a client scheduled for a routine glycosylated hemoglobin (HbA1c) test?

💡 Hint

Reflect on the appropriate activities and level of exertion recommended for a patient during their recovery from cataract surgery.

36 / 50

36. Which comment made by a patient recovering from cataract surgery suggests to the nurse that further instruction is required?

💡 Hint

Insulin is a hormone that plays a crucial role in managing blood sugar levels. Consider its primary action in relation to glucose utilization by cells.

37 / 50

37. What is the main function of insulin in the body?

💡 Hint

Acute pancreatitis presents with specific symptoms related to the pancreas' role in digestion and the inflammation it causes. Focus on the signs that reflect these functions and consequences.

38 / 50

38. Which assessment findings would a nurse typically observe in a patient with acute pancreatitis?

💡 Hint

Consider the purpose of the cuff on an endotracheal tube and how a leak might affect the patient's ability to perform certain actions involving air movement through the vocal cords.

39 / 50

39. Which nursing observation suggests that the cuff on an endotracheal tube is leaking?

💡 Hint

This question focuses on an early warning sign related to low blood sugar. Consider how the body might respond to a sudden decrease in glucose levels in order to maintain homeostasis.

40 / 50

40. Which symptom is an early indication of hypoglycemia that a nurse should be aware of in order to identify an insulin reaction?

💡 Hint

Vasopressin, also known as antidiuretic hormone, plays a role in regulating water balance in the body. Consider which evaluation would be relevant to this function.

41 / 50

41. When assessing a client on vasopressin therapy, what should the nurse evaluate to determine the therapeutic response to this medication?

💡 Hint

Consider the peak action times of both regular and NPH insulin. Regular insulin typically peaks around 2-4 hours after administration, while NPH insulin peaks around 6-8 hours after administration.

42 / 50

42. In a diabetic patient who receives a mixture of regular and NPH insulin at 7:00 AM, at which two time intervals should the patient be particularly vigilant for symptoms of hypoglycemia?

💡 Hint

When a patient is comatose and hypoglycemic, consider the most immediate and effective method to raise their blood sugar levels.

43 / 50

43. A comatose and hypoglycemic patient has a blood sugar level of 50 mg/dl. Which nursing intervention should be prioritized?

💡 Hint

Retinal detachment can be caused by eye injuries, which are more common in individuals who participate in physical contact sports, as they are at a higher risk of sustaining trauma to the eye.

44 / 50

44. Which client has the greatest risk of experiencing a retinal detachment?

💡 Hint

When considering the effects of COPD on a patient's daily life, think about how the need for continuous oxygen therapy may impact their level of activity at home.

45 / 50

45. A client with COPD is being discharged home on continuous oxygen at a rate of 2 L/min via a nasal cannula. What information should the nurse provide to the client and his wife regarding home oxygen use?

💡 Hint

Consider the importance of timely administration of blood products and the potential risks associated with delays in the process.

46 / 50

46. When administering one unit of whole blood to a client, which nursing action is appropriate?

💡 Hint

Consider the primary manifestations of polycythemia vera, which are associated with increased red blood cell production and blood viscosity.

47 / 50

47. When evaluating a client diagnosed with polycythemia vera, which characteristics should the nurse expect to observe?

💡 Hint

Consider the potential hereditary aspects of glaucoma and the importance of regular screening for at-risk individuals.

48 / 50

48. A patient expresses concern about glaucoma being hereditary and is worried about her adult children. What is the most appropriate response?

💡 Hint

Consider the primary treatment for pernicious anemia, which involves addressing the deficiency of a specific vitamin essential for red blood cell production.

49 / 50

49. A patient diagnosed with pernicious anemia is receiving discharge instructions from the nurse. What should the nurse teach the client about the medication they will need to take at home?

💡 Hint

In acute-angle glaucoma, it is crucial to avoid medications that could potentially worsen the condition by dilating the pupil, further increasing intraocular pressure.

50 / 50

50. For a client with acute-angle glaucoma, which medication order would the nurse question?

Exam Mode

Welcome to your Medical-Surgical Nursing Exam 4! This exam is carefully designed to provide you with a realistic test-taking experience, preparing you for the pressures of an actual nursing exam.

 

Exam Details

  • Number of Questions: 50 items
  • Mode: Exam Mode

Exam Instructions

  1. Exam Mode: This mode is intended to simulate the environment of an actual exam. Questions and choices will be presented one at a time.
  2. Time Limit: Each question must be answered within 90 seconds. The entire exam should be completed within 75 minutes.
  3. Feedback and Grading: Upon completion of the exam, you will be able to see your grade and the correct answers to all questions. This will allow you to evaluate your performance and understand areas for improvement.

Tips For Success

  • Read each question carefully. You have 90 seconds per question, so make sure you understand the question before selecting your answer.
  • Pace yourself. Remember, you have 75 minutes in total, so try to maintain a steady rhythm.
  • Focus on one question at a time. Try not to worry about the questions to come.
  • Stay calm under pressure. Use your knowledge and trust your instincts.
  • Remember, it's not just about the score, but about the learning process.

This exam is not only a measurement of your current understanding, but also a valuable learning tool to prepare you for your future nursing career. Click 'Start Exam' when you're ready to begin. Good luck!

1 / 50

1. Which individual has the highest probability of developing iron deficiency anemia?

2 / 50

2. The spouse of a patient with COPD is concerned about providing care at home. Which statement by the nurse offers the most accurate information?

3 / 50

3. For a client with acute-angle glaucoma, which medication order would the nurse question?

4 / 50

4. Upon anesthetizing a client's eye, which crucial guidelines should the nurse provide to the client?

5 / 50

5. In a diabetic patient who receives a mixture of regular and NPH insulin at 7:00 AM, at which two time intervals should the patient be particularly vigilant for symptoms of hypoglycemia?

6 / 50

6. For a patient with elevated intracranial pressure (ICP), the nurse should be cautious about administering which type of eye drop?

7 / 50

7. What is the most important instruction for a patient prescribed daily levothyroxine (Synthroid) regarding the administration of the drug?

8 / 50

8. The nurse should educate the client about metformin (Glucophage), emphasizing that it:

9 / 50

9. A patient diagnosed with type 2 diabetes has been prescribed prednisone to alleviate severe arthritis pain. What is a potential change that necessitates careful monitoring by the nurse?

10 / 50

10. While caring for a client receiving a blood transfusion, the transfusion was initiated 30 minutes ago at a rate of 100 ml/hr. The client starts complaining of lower back pain, headache, and increased restlessness. What should be the nurse's first course of action?

11 / 50

11. When assessing a client on vasopressin therapy, what should the nurse evaluate to determine the therapeutic response to this medication?

12 / 50

12. Which symptom is an early indication of hypoglycemia that a nurse should be aware of in order to identify an insulin reaction?

13 / 50

13. When assessing a 25-year-old woman presenting with dizziness, weakness, and palpitations, what is the initial priority for the nurse to evaluate while obtaining her health history?

14 / 50

14. What is an important aspect to include in the nursing care of a patient with angle-closure glaucoma?

15 / 50

15. A patient diagnosed with pernicious anemia is receiving discharge instructions from the nurse. What should the nurse teach the client about the medication they will need to take at home?

16 / 50

16. Which comment made by a patient recovering from cataract surgery suggests to the nurse that further instruction is required?

17 / 50

17. A comatose and hypoglycemic patient has a blood sugar level of 50 mg/dl. Which nursing intervention should be prioritized?

18 / 50

18. When discharging a child with leukemia who has just started chemotherapy, what guidance should the nurse include in the teaching plan for the child's parents?

19 / 50

19. A client contacts the nurse after accidentally splashing an unknown chemical into their eyes. What is the most crucial action the nurse should advise the client to take immediately?

20 / 50

20. Which medication should a nurse expect to administer to a client diagnosed with Meniere's disease?

21 / 50

21. On the first postoperative day following a right lower lobe (RLL) lobectomy, the client breathes and coughs but struggles to bring up mucus. What indicates that the client is not effectively clearing secretions?

22 / 50

22. When evaluating a client diagnosed with polycythemia vera, which characteristics should the nurse expect to observe?

23 / 50

23. What is the underlying cause of exophthalmos in a patient with thyroid disease?

24 / 50

24. For a client in the ICU diagnosed with disseminated intravascular coagulation (DIC), the nurse should anticipate administering which fluid?

25 / 50

25. A patient experiencing a sickle cell crisis is admitted to the emergency department. What should be the priorities of care for this client?

26 / 50

26. What should the nurse instruct a diabetic patient about the role of exercise in their treatment program?

27 / 50

27. What is the priority nursing diagnosis for a patient admitted to the hospital with a diagnosis of diabetes insipidus?

28 / 50

28. Which client has the greatest risk of experiencing a retinal detachment?

29 / 50

29. While caring for a patient experiencing a thyroid crisis, which order should the nurse question?

30 / 50

30. When educating a client and their family about using a hearing aid, on which key piece of information will the nurse base the teaching?

31 / 50

31. What assessment finding should a nurse expect in a patient who has been receiving long-term steroid therapy?

32 / 50

32. What is the main function of insulin in the body?

33 / 50

33. When a client experiences vertigo while walking down the hall, what is the priority nursing action?

34 / 50

34. What measures should be taken in postoperative nursing care following a thyroidectomy?

35 / 50

35. Which assessment findings would a nurse typically observe in a patient with acute pancreatitis?

36 / 50

36. Which statement accurately characterizes the process of suctioning through an endotracheal tube?

37 / 50

37. What medication should the nurse have on hand for the emergency treatment of tetany in a patient who has undergone a thyroidectomy?

38 / 50

38. How can a nurse determine if a patient receiving 20 units of NPH insulin subcutaneously at 7:00 AM daily is experiencing a hypoglycemic reaction at 3:00 PM?

39 / 50

39. Which nursing observation suggests that the cuff on an endotracheal tube is leaking?

40 / 50

40. What is the primary goal of a myringotomy procedure that the nurse should discuss with the parents of a child scheduled for the surgery?

41 / 50

41. Before administering a unit of packed red blood cells, the nurse is preparing to initiate an IV infusion. Which fluid should the nurse choose to maintain the infusion prior to hanging the blood unit?

42 / 50

42. What essential information should the nurse provide to a client scheduled for a routine glycosylated hemoglobin (HbA1c) test?

43 / 50

43. When admitting a patient with Meniere's disease, which assessment is of the utmost importance for the nurse to conduct?

44 / 50

44. A client with COPD is being discharged home on continuous oxygen at a rate of 2 L/min via a nasal cannula. What information should the nurse provide to the client and his wife regarding home oxygen use?

45 / 50

45. A patient expresses concern about glaucoma being hereditary and is worried about her adult children. What is the most appropriate response?

46 / 50

46. Which statement demonstrates that a child diagnosed with conjunctivitis has comprehended the nurse's instructions?

47 / 50

47. What laboratory values would a nurse expect to find when assessing a patient with Syndrome of Inappropriate Antidiuretic Hormone (SIADH)?

48 / 50

48. When administering one unit of whole blood to a client, which nursing action is appropriate?

49 / 50

49. Following an adrenalectomy for a client diagnosed with pheochromocytoma, the nurse should meticulously evaluate this patient for:

50 / 50

50. In order to ensure the safety and well-being of a patient following a stapedectomy, which aspect should be incorporated into the nursing care plan?

Text Mode

Text Mode – Text version of the exam

Questions

1. What measures should be taken in postoperative nursing care following a thyroidectomy?

A. Utilize a gentle cervical collar to limit neck motion.
B. Encourage the patient to speak every 5-10 minutes if hoarseness occurs.
C. Inspect the dressing on the back of the neck for any signs of bleeding.
D. Administer diuretics to reduce fluid retention.

2. Which assessment findings would a nurse typically observe in a patient with acute pancreatitis?

A. Steatorrhea, abdominal pain, and fever.
B. Ascites, spider angiomas, and gynecomastia.
C. Melena, continuous vomiting, and increased bowel sounds.
D. Fever, low blood sugar levels, and diabetic hyperosmolar nonketotic syndrome (DHN).

3.) How can a nurse determine if a patient receiving 20 units of NPH insulin subcutaneously at 7:00 AM daily is experiencing a hypoglycemic reaction at 3:00 PM?

A. Assess the patient’s breath for an acetone odor.
B. Examine the patient and their bedding for dampness.
C. Monitor the patient for Kussmaul respirations.
D. Evaluate the patient’s pupils for dilation.

4.) A comatose and hypoglycemic patient has a blood sugar level of 50 mg/dl. Which nursing intervention should be prioritized?

A. Provide 50% glucose intravenously.
B. Offer the patient a high-carbohydrate meal.
C. Deliver 1000 ml of D5W over a span of 12 hours.
D. Examine the patient’s urine for sugar and acetone content.

5.) What medication should the nurse have on hand for the emergency treatment of tetany in a patient who has undergone a thyroidectomy?

A. Magnesium sulfate
B. Calcium chloride
C. Magnesium citrate
D. Potassium chloride

6.) What should the nurse instruct a diabetic patient about the role of exercise in their treatment program?

A. Increased activity leads to greater carbohydrate usage, necessitating more insulin.
B. Exercise may temporarily raise blood sugar levels, but overall it helps in stabilizing them.
C. Exercise causes the body to use carbohydrates for energy, reducing insulin requirements.
D. Increased activity promotes insulin utilization, so the patient should decrease their carbohydrate intake.

7.) What is the underlying cause of exophthalmos in a patient with thyroid disease?

A. Retro-orbital tissue edema, causing the eyes to protrude.
B. Compromised vision, leading the patient to squint for better sight.
C. Diminished extraocular eye movements, resulting in a “thyroid stare”.
D. Inflammation of the eye muscles, leading to protrusion.

8.) What essential information should the nurse provide to a client scheduled for a routine glycosylated hemoglobin (HbA1c) test?

A. Consume only water after midnight and arrive at the clinic early in the morning.
B. Have a regular breakfast and be at the clinic 2 hours ahead due to multiple blood draws.
C. Anticipate spending several hours at the clinic due to multiple blood draws.
D. Visit the clinic at the earliest opportunity for the blood draw.

9. When assessing a client on vasopressin therapy, what should the nurse evaluate to determine the therapeutic response to this medication?

A. Urine specific gravity
B. Serum electrolyte levels
C. Oxygen saturation levels
D. Blood glucose

10. What is the main function of insulin in the body?

A. Facilitates glucose transportation across cell membranes.
B. Supports gluconeogenesis.
C. Stimulates pancreatic beta cells.
D. Reduces intestinal glucose absorption.

11. Which symptom is an early indication of hypoglycemia that a nurse should be aware of in order to identify an insulin reaction?

A. Excessive sweating
B. Sleepiness
C. Intense thirst
D. High blood pressure

12. A patient diagnosed with type 2 diabetes has been prescribed prednisone to alleviate severe arthritis pain. What is a potential change that necessitates careful monitoring by the nurse?

A. Elevated blood glucose levels.
B. Enhanced creatinine clearance.
C. Increased platelet clumping.
D. Increased insulin sensitivity.

13. In a diabetic patient who receives a mixture of regular and NPH insulin at 7:00 AM, at which two time intervals should the patient be particularly vigilant for symptoms of hypoglycemia?

A. Between 12:00 PM and 1:00 PM
B. At 10:00 AM and 10:00 PM
C. At 11:00 AM and 5:00 PM
D. Between 2:00 PM and 4:00 PM

14. The nurse should educate the client about metformin (Glucophage), emphasizing that it:

A. Ought to be consumed alongside meals.
B. May result in weight gain.
C. Has the potential to enhance aspirin’s effects.
D. Is recommended for intake during nighttime.

15. What is the priority nursing diagnosis for a patient admitted to the hospital with a diagnosis of diabetes insipidus?

A. Fluid imbalance: risk for deficient fluid volume.
B. Sleep pattern disturbance due to nocturia.
C. Activity intolerance related to muscle weakness.
D. Risk for impaired skin integrity related to generalized edema.

16. Following an adrenalectomy for a client diagnosed with pheochromocytoma, the nurse should meticulously evaluate this patient for:

A. Significant variations in blood pressure.
B. Low potassium levels (hypokalemia).
C. Excessive sodium and water intake.
D. High blood sugar (hyperglycemia).

17. What assessment finding should a nurse expect in a patient who has been receiving long-term steroid therapy?

A. Central obesity
B. Jaundice
C. Flank pain
D. Diminished appetite.

18. What laboratory values would a nurse expect to find when assessing a patient with Syndrome of Inappropriate Antidiuretic Hormone (SIADH)?

A. Serum sodium (Na) = 120 mEq/L and low serum osmolality
B. Serum potassium (K) = 5 mEq/L and low serum osmolality
C. Serum sodium (Na) = 150 mEq/L and low urine osmolality
D. Serum potassium (K) = 3 mEq/L and high serum osmolality

19. While caring for a patient experiencing a thyroid crisis, which order should the nurse question?

A. Hyperthermia blanket
B. Intravenous fluid
C. Propylthiouracil
D. Propranolol (Inderal)

20. What is the most important instruction for a patient prescribed daily levothyroxine (Synthroid) regarding the administration of the drug?

A. Contact the physician immediately if palpitations or nervousness occur.
B. Reduce the consumption of juices and fruits high in potassium and calcium.
C. Take the medication at bedtime to minimize nausea and gas side effects.
D. Only take the medication when experiencing symptoms.

21. A patient expresses concern about glaucoma being hereditary and is worried about her adult children. What is the most appropriate response?

A. “There’s no need to worry; glaucoma is not a hereditary condition.”
B. “Glaucoma screening should be part of an annual eye exam for everyone over the age of 50.”
C. “Glaucoma may have a genetic component, and your children over 30 years old should be screened yearly.”
D. “Glaucoma is only hereditary if it develops before age 40, so your children are unlikely to be affected.”

22. What is an important aspect to include in the nursing care of a patient with angle-closure glaucoma?

A. Evaluate medications to identify any that may cause an increase in intraocular pressure (IOP) as a side effect.
B. Assess the patient’s level of discomfort, as they may experience significant pain until the optic nerve atrophies.
C. Control blood pressure to reduce the potential loss of peripheral vision.
D. Apply pressure to the patient’s closed eyes to alleviate intraocular pressure.

23. What is the primary goal of a myringotomy procedure that the nurse should discuss with the parents of a child scheduled for the surgery?

A. Equalize pressure on the tympanic membrane.
B. Facilitate drainage from the ear.
C. Cleanse the Eustachian tube.
D. Eliminate excess earwax buildup in the ear canal.

24. Upon anesthetizing a client’s eye, which crucial guidelines should the nurse provide to the client?

A. Refrain from rubbing the eye for a duration of 15-20 minutes.
B. Use sunglasses while exposed to direct sunlight for the following 6 hours.
C. Perform hourly eye irrigation to avert dryness.
D. Immediately wash the eye with tap water for 5 minutes.

25. Which statement demonstrates that a child diagnosed with conjunctivitis has comprehended the nurse’s instructions?

A. “I should wear contact lenses to help my eyes heal faster.”
B. “Upon waking up, I can remove the crusty debris from my eyelashes using my fingers.”
C. “While my eyes are infected, I will use my personal washcloth and towel.”
D. “Gently rubbing my eye with the back of my hand is acceptable.”

26. Which medication should a nurse expect to administer to a client diagnosed with Meniere’s disease?

A. Hydrochlorothiazide (HydroDIURIL)
B. Nifedipine
C. Propranolol
D. Acetaminophen

27. For a client with acute-angle glaucoma, which medication order would the nurse question?

A. Atropine (Atropisol) 1-2 drops in each eye immediately.
B. Hydrochlorothiazide (Diuril) 25 mg PO daily.
C. Carbachol (Isopto Carbachol) eye drops; 1 drop 2 times a day.
D. Warfarin (Coumadin) 5 mg PO daily.

28. When educating a client and their family about using a hearing aid, on which key piece of information will the nurse base the teaching?

A. Offers mechanical support for impaired parts of the ear.
B. Enhances sound volume without necessarily improving hearing ability.
C. Stimulates the inner ear’s neural network to boost sound.
D. Repairs damage to the auditory nerve.

29. When a client experiences vertigo while walking down the hall, what is the priority nursing action?

A. Help the client to either sit or lie down.
B. Determine whether the sensation is vertigo or dizziness.
C. Urge the client to continue walking to see if symptoms dissipate.
D. Instruct the client to sit in a chair and lower their head.

30. Which client has the greatest risk of experiencing a retinal detachment?

A. 4-year-old with amblyopia.
B. 17-year-old engaged in physical contact sports.
C. 33-year-old with severe ptosis and diplopia.
D. 72-year-old with nystagmus and Bell’s palsy.

31. In order to ensure the safety and well-being of a patient following a stapedectomy, which aspect should be incorporated into the nursing care plan?

A. Initiate measures to prevent falls.
B. Take steps to avoid aspiration.
C. Provide 2-4L/min of oxygen through a nasal cannula.
D. Give sedatives to encourage rest and relaxation.

32. Which comment made by a patient recovering from cataract surgery suggests to the nurse that further instruction is required?

A. “I’ll contact you if I experience considerable pain.”
B. “I’ll keep taking my Metamucil for one more week.”
C. “If it’s recommended that I rest at home and limit my activities, I’ll follow that advice.”
D. “I’ll opt for doing laundry this afternoon rather than going to work.”

33. For a patient with elevated intracranial pressure (ICP), the nurse should be cautious about administering which type of eye drop?

A. Artificial tears
B. Betaxolol (Betoptic)
C. Acetazolamide (Diamox)
D. Epinephrine HCL (Epirate)

34. When admitting a patient with Meniere’s disease, which assessment is of the utmost importance for the nurse to conduct?

A. Immunization history.
B. Preliminary hearing evaluation.
C. Dietary history.
D. Regularity and intensity of symptoms.

35. When assessing a 25-year-old woman presenting with dizziness, weakness, and palpitations, what is the initial priority for the nurse to evaluate while obtaining her health history?

A. Dietary habits.
B. Coping mechanisms and stress tolerance.
C. Activity and exercise routines.
D. Family health status.

36. When discharging a child with leukemia who has just started chemotherapy, what guidance should the nurse include in the teaching plan for the child’s parents?

A. Administer aspirin for pain or fever.
B. Offer a diet with low protein and high carbohydrates.
C. Encourage the use of humidifiers throughout the home.
D. Refrain from providing uncooked or unpeeled fresh vegetables.

37. Which individual has the highest probability of developing iron deficiency anemia?

A. A toddler primarily consumes milk as a source of nutrition.
B. A cancer patient undergoing radiation therapy twice weekly.
C. A 15-year-old patient experiencing a sickle cell crisis.
D. A patient with chronic respiratory disease receiving long-term corticosteroid treatment.

38. When administering one unit of whole blood to a client, which nursing action is appropriate?

A. Observe the client’s vital signs during the initial 5 minutes.
B. Begin the transfusion within 30 minutes of obtaining the blood.
C. Start an IV with 5% dextrose in water (D5W) to ensure a patent access site.
D. Infuse the blood quickly to avoid clot formation.

39. A client contacts the nurse after accidentally splashing an unknown chemical into their eyes. What is the most crucial action the nurse should advise the client to take immediately?

A. Flush the eye thoroughly with water or saline solution.
B. Place a pad soaked in sterile saline solution over the eye.
C. Visit the nearest emergency room.
D. Rub the affected eye vigorously to eliminate debris or chemicals.

40. While caring for a client receiving a blood transfusion, the transfusion was initiated 30 minutes ago at a rate of 100 ml/hr. The client starts complaining of lower back pain, headache, and increased restlessness. What should be the nurse’s first course of action?

A. Decelerate the infusion, assess vital signs, and review the client’s history of transfusion reactions.
B. Halt the transfusion, detach the blood tubing, and commence a primary infusion of normal saline solution.
C. Double-check the blood unit for accurate identification numbers and cross-match details.
D. Give the client a dose of diuretic medication.

41. Before administering a unit of packed red blood cells, the nurse is preparing to initiate an IV infusion. Which fluid should the nurse choose to maintain the infusion prior to hanging the blood unit?

A. 0.9% NaCl
B. 10% dextrose in water (D10W)
C. Lactated Ringer’s solution
D. D5W/0.45% NaCl

42. For a client in the ICU diagnosed with disseminated intravascular coagulation (DIC), the nurse should anticipate administering which fluid?

A. Fresh Frozen Plasma (FFP)
B. Whole blood
C. Packed Red Blood Cells
D. Potassium chloride

43. When evaluating a client diagnosed with polycythemia vera, which characteristics should the nurse expect to observe?

A. Headaches, dyspnea, and claudication.
B. Elevated fatigue and bleeding tendencies.
C. Hemoglobin levels below 13 mg/dl.
D. Low platelet count and frequent infections.

44. A patient diagnosed with pernicious anemia is receiving discharge instructions from the nurse. What should the nurse teach the client about the medication they will need to take at home?

A. Monthly vitamin B12 injections will be required.
B. Reduce consumption of leafy green vegetables due to elevated vitamin K levels.
C. Consume daily iron supplements to enhance red blood cell production.
D. Coagulation studies are essential for evaluating medications.

45. A patient experiencing a sickle cell crisis is admitted to the emergency department. What should be the priorities of care for this client?

A. Hydration, oxygenation, pain management.
B. Nutrition, hydration, electrolyte balance.
C. Hydration, pain management, electrolyte balance.
D. Hydration, oxygenation, electrolyte balance.

46. Which nursing observation suggests that the cuff on an endotracheal tube is leaking?

A. Increased crackles (rales) in the left lung field.
B. Decreased pulse rate.
C. The ability of the client to speak.
D. Wheezing detected upon auscultation.

47. On the first postoperative day following a right lower lobe (RLL) lobectomy, the client breathes and coughs but struggles to bring up mucus. What indicates that the client is not effectively clearing secretions?

A. Chest X-ray reveals right-sided pleural fluid.
B. Decreased heart rate.
C. PCO2 levels rise from 35 to 45 mm Hg.
D. A few scattered crackles in RLL detected during auscultation.

48. A client with COPD is being discharged home on continuous oxygen at a rate of 2 L/min via a nasal cannula. What information should the nurse provide to the client and his wife regarding home oxygen use?

A. The client can discontinue oxygen usage once he feels better.
B. Oxygen use will completely alleviate the client’s shortness of breath.
C. The client will need to limit activity at home due to the requirement for oxygen.
D. Oxygen therapy can cure COPD.

49. Which statement accurately characterizes the process of suctioning through an endotracheal tube?

A. The catheter is placed within the endotracheal tube and remains there for ongoing suctioning.
B. The catheter is initially inserted through the nose to suction the upper airway, then removed and inserted into the endotracheal tube to suction the lower airway.
C. The catheter is introduced into the endotracheal tube until resistance is encountered, with intermittent suction applied during withdrawal.
D. Suctioning via an endotracheal tube is not required.

50. The spouse of a patient with COPD is concerned about providing care at home. Which statement by the nurse offers the most accurate information?

A. “Your husband may experience shortness of breath during activities, but this won’t hurt him.”
B. “Organize a routine where your husband completes essential tasks before noon, allowing him to rest in the afternoon and evening.”
C. “It’s best to avoid emotionally charged situations that can exacerbate his shortness of breath.”
D. “Your husband’s shortness of breath will inevitably worsen over time, and there is no way to prevent it.”

 

Answers and Rationales

1. Correct answer:

C. Inspect the dressing on the back of the neck for any signs of bleeding. After a thyroidectomy, it is essential to monitor the surgical site for any signs of bleeding. Inspecting the dressing on the back of the neck helps identify bleeding or hematoma formation, which can lead to complications such as airway obstruction or infection. Prompt recognition and intervention are crucial in managing these complications.

Incorrect answer options:

A. Utilize a gentle cervical collar to limit neck motion. Using a cervical collar is not typically necessary after a thyroidectomy. Instead, the patient should be encouraged to perform gentle neck exercises and range-of-motion activities as tolerated to maintain mobility and prevent stiffness.

B. Encourage the patient to speak every 5-10 minutes if hoarseness occurs. Hoarseness may occur after a thyroidectomy due to injury or irritation of the laryngeal nerves. Encouraging the patient to speak every 5-10 minutes is not necessary and may even exacerbate the irritation. The patient should be advised to rest their voice, avoid straining or coughing, and use a humidifier to moisten the air. If hoarseness persists, the patient should be evaluated by a healthcare provider.

D. Administer diuretics to reduce fluid retention. Administering diuretics to reduce fluid retention is not a standard postoperative nursing intervention following a thyroidectomy. Instead, it is important to monitor the patient’s fluid balance, maintain adequate hydration, and assess for any signs of fluid overload or electrolyte imbalances, such as hypocalcemia, which may result from inadvertent removal or damage to the parathyroid glands during surgery.

2. Correct answer:

A. Steatorrhea, abdominal pain, and fever. A patient with acute pancreatitis typically presents with steatorrhea (fatty stools), abdominal pain, and fever. The abdominal pain is usually severe and sudden, located in the epigastric region, and may radiate to the back. Other symptoms may include nausea, vomiting, tachycardia, and hypotension.

Incorrect answer options:

B. Ascites, spider angiomas, and gynecomastia. These findings are more commonly associated with cirrhosis of the liver. Ascites is the accumulation of fluid in the abdominal cavity, spider angiomas are dilated blood vessels on the skin, and gynecomastia refers to the enlargement of male breast tissue due to hormonal imbalance.

C. Melena, continuous vomiting, and increased bowel sounds. Melena (dark, tarry stools) and continuous vomiting are not typical findings in acute pancreatitis. Increased bowel sounds are also not characteristic of acute pancreatitis, as patients may actually have decreased bowel sounds due to paralytic ileus (lack of intestinal movement) secondary to inflammation.

D. Fever, low blood sugar levels, and diabetic hyperosmolar nonketotic syndrome (DHN). Although fever is a common symptom in acute pancreatitis, low blood sugar levels and diabetic hyperosmolar nonketotic syndrome (DHN) are not typical findings in this condition. DHN is a serious complication of diabetes mellitus characterized by severe hyperglycemia, dehydration, and electrolyte imbalances.

3. Correct answer:

B. Examine the patient and their bedding for dampness. A hypoglycemic reaction, or low blood sugar, can cause a patient to sweat excessively, leading to dampness on their skin and bedding. In this scenario, assessing the patient and their bedding for dampness can provide a clue that the patient may be experiencing hypoglycemia at 3:00 PM.

Incorrect answer options:

A. Assess the patient’s breath for an acetone odor. An acetone odor on the breath is a sign of diabetic ketoacidosis (DKA), which is associated with high blood sugar levels, not hypoglycemia.

C. Monitor the patient for Kussmaul respirations. Kussmaul respirations are rapid, deep breaths that can be seen in patients with diabetic ketoacidosis (DKA) due to the body’s attempt to eliminate excess carbon dioxide and compensate for metabolic acidosis. This is not a typical sign of hypoglycemia.

D. Evaluate the patient’s pupils for dilation. Pupil dilation is not a specific sign of hypoglycemia. Signs of hypoglycemia include weakness, shakiness, dizziness, confusion, irritability, hunger, headache, and sweating. In severe cases, hypoglycemia may cause seizures, unconsciousness, or coma.

4.Correct answer:

A. Provide 50% glucose intravenously. In a comatose and hypoglycemic patient with a blood sugar level of 50 mg/dl, the priority nursing intervention is to provide 50% glucose intravenously. This intervention quickly raises the blood sugar level and helps to prevent further complications associated with severe hypoglycemia, such as brain damage or seizures.

Incorrect answer options:

B. Offer the patient a high-carbohydrate meal. A comatose patient cannot safely consume a meal, as they are at a high risk for aspiration. Moreover, oral administration of carbohydrates would not raise the blood sugar level quickly enough in a severely hypoglycemic patient.

C. Deliver 1000 ml of D5W over a span of 12 hours. Administering 1000 ml of D5W (5% dextrose in water) over 12 hours would not raise the patient’s blood sugar level rapidly enough to address the immediate needs of a severely hypoglycemic patient.

D. Examine the patient’s urine for sugar and acetone content. While examining the patient’s urine for sugar and acetone content may provide information about the patient’s overall metabolic status, it is not a priority intervention for a comatose and hypoglycemic patient. Immediate treatment of hypoglycemia should be prioritized to prevent further complications.

5. Correct answer:

B. Calcium chloride. In a patient who has undergone a thyroidectomy, the emergency treatment for tetany is calcium chloride. Tetany can occur due to hypocalcemia, which is a result of inadvertent damage or removal of the parathyroid glands during the surgical procedure. Calcium chloride helps to rapidly raise serum calcium levels, providing relief from the symptoms of tetany.

Incorrect answer options:

A. Magnesium sulfate. Magnesium sulfate is not used for the emergency treatment of tetany. It is commonly used to treat and prevent seizures in pregnant women with eclampsia or severe preeclampsia.

C. Magnesium citrate. Magnesium citrate is not used for the emergency treatment of tetany. It is an osmotic laxative used to treat constipation or to cleanse the bowel before a medical procedure.

D. Potassium chloride. Potassium chloride is not used for the emergency treatment of tetany. It is primarily used to treat or prevent hypokalemia (low potassium levels) in patients who have lost potassium due to illness or medication use.

6. Correct answer:

C. Exercise causes the body to use carbohydrates for energy, reducing insulin requirements. Exercise plays an essential role in the management of diabetes. It helps the body use carbohydrates for energy, which can reduce insulin requirements. Regular physical activity can also improve insulin sensitivity, promote weight loss, and help maintain overall cardiovascular health. The nurse should instruct diabetic patients to engage in regular exercise as part of their treatment program, but they should also be advised to monitor their blood glucose levels before and after exercise to ensure appropriate adjustments are made to their insulin or carbohydrate intake.

Incorrect answer options:

A. Increased activity leads to greater carbohydrate usage, necessitating more insulin. This statement is not accurate. While increased activity can lead to greater carbohydrate usage, it actually reduces insulin requirements as the body becomes more efficient at utilizing carbohydrates for energy.

B. Exercise may temporarily raise blood sugar levels, but overall it helps in stabilizing them. Exercise generally lowers blood sugar levels by increasing the body’s sensitivity to insulin and promoting carbohydrate usage. In some cases, intense exercise may cause a temporary increase in blood sugar levels; however, this is not the typical response.

D. Increased activity promotes insulin utilization, so the patient should decrease their carbohydrate intake. While increased activity does promote insulin utilization, it does not necessarily mean that a patient should decrease their carbohydrate intake. Instead, patients should monitor their blood sugar levels and adjust their insulin or carbohydrate intake accordingly, in consultation with their healthcare provider.

7. Correct answer:

A. Retro-orbital tissue edema, causing the eyes to protrude. Exophthalmos, or protrusion of the eyes, is commonly seen in patients with Graves’ disease, an autoimmune thyroid disorder. The underlying cause is retro-orbital tissue edema, which leads to an increase in the volume of the tissues behind the eye. This causes the eyeballs to protrude forward, resulting in exophthalmos.

Incorrect answer options:

B. Compromised vision, leading the patient to squint for better sight. Compromised vision may be a symptom of thyroid eye disease, but it is not the underlying cause of exophthalmos. Squinting for better sight does not result in the protrusion of the eyes.

C. Diminished extraocular eye movements, resulting in a “thyroid stare”. Diminished extraocular eye movements can be a symptom of thyroid eye disease, but they are not the cause of exophthalmos. The “thyroid stare” is a term used to describe the wide-eyed appearance of a patient with exophthalmos, but it is not the underlying cause.

D. Inflammation of the eye muscles, leading to protrusion. Inflammation of the eye muscles can occur in thyroid eye disease, but it is not the primary cause of exophthalmos. The main cause is retro-orbital tissue edema, which pushes the eyes forward and leads to protrusion.

8. Correct answer:

D. Visit the clinic at the earliest opportunity for the blood draw. A glycosylated hemoglobin (HbA1c) test measures the average blood sugar levels over the past 2-3 months. The test does not require fasting or any specific preparation. The nurse should instruct the client to visit the clinic at the earliest opportunity for the blood draw, as the test can be performed at any time of day, regardless of when the client last ate.

Incorrect answer options:

A. Consume only water after midnight and arrive at the clinic early in the morning. Fasting is not required for an HbA1c test. The client does not need to avoid eating or drinking anything other than water after midnight.

B. Have a regular breakfast and be at the clinic 2 hours ahead due to multiple blood draws. The HbA1c test requires only one blood draw, so the client does not need to arrive at the clinic 2 hours ahead of time.

C. Anticipate spending several hours at the clinic due to multiple blood draws. As mentioned earlier, the HbA1c test requires only one blood draw, and the client does not need to spend several hours at the clinic.

9. Correct answer:

A. Urine specific gravity. Vasopressin, also known as antidiuretic hormone (ADH), is used to treat diabetes insipidus or certain conditions resulting in low blood pressure. Vasopressin helps the kidneys retain water, which reduces urine output and increases urine concentration. When assessing a client on vasopressin therapy, the nurse should evaluate urine specific gravity to determine the therapeutic response to this medication. An increase in urine specific gravity indicates a more concentrated urine, which suggests a positive therapeutic response to vasopressin.

Incorrect answer options:

B. Serum electrolyte levels. While monitoring serum electrolyte levels is essential for many patients, it is not the primary indicator of the therapeutic response to vasopressin. Urine specific gravity is a more direct indicator of the medication’s effectiveness in this case.

C. Oxygen saturation levels. Oxygen saturation levels are not directly related to the therapeutic response of vasopressin. Monitoring oxygen saturation levels is important in many clinical situations but is not the primary indicator of vasopressin effectiveness.

D. Blood glucose. Blood glucose levels are not directly related to the therapeutic response of vasopressin. Monitoring blood glucose levels is essential for patients with diabetes or other conditions affecting glucose metabolism, but it is not the primary indicator of vasopressin effectiveness.

10. Correct answer:

A. Facilitates glucose transportation across cell membranes. Insulin is a hormone produced by the beta cells of the pancreas. Its main function is to facilitate the transportation of glucose across cell membranes, allowing glucose to enter the cells and be used for energy production. By doing so, insulin helps regulate blood sugar levels and maintains overall glucose homeostasis in the body.

Incorrect answer options:

B. Supports gluconeogenesis. Gluconeogenesis is the process of creating glucose from non-carbohydrate sources, such as amino acids and glycerol. Insulin does not support gluconeogenesis; instead, it counteracts this process by promoting glucose uptake and utilization in cells.

C. Stimulates pancreatic beta cells. Insulin does not stimulate pancreatic beta cells; it is produced by these cells. Pancreatic beta cells are stimulated by an increase in blood glucose levels, which signals the need for insulin secretion to help regulate blood sugar levels.

D. Reduces intestinal glucose absorption. Insulin does not directly reduce intestinal glucose absorption. Its primary function is to facilitate glucose transportation across cell membranes, helping cells utilize glucose for energy production.

11. Correct answer:

A. Excessive sweating. Excessive sweating is an early indication of hypoglycemia (low blood sugar) that a nurse should be aware of to identify an insulin reaction. Hypoglycemia can occur in diabetic patients who have taken too much insulin, have not eaten enough, or have engaged in excessive physical activity. Other early symptoms of hypoglycemia include shakiness, dizziness, rapid heartbeat, and hunger.

Incorrect answer options:

B. Sleepiness. Sleepiness is not an early indication of hypoglycemia or an insulin reaction. However, it can be a symptom of hyperglycemia (high blood sugar).

C. Intense thirst. Intense thirst is a symptom of hyperglycemia, not hypoglycemia. When blood sugar levels are high, the body tries to eliminate excess glucose through increased urination, which can lead to dehydration and intense thirst.

D. High blood pressure. High blood pressure is not an early indication of hypoglycemia or an insulin reaction. While high blood pressure can be associated with diabetes and its complications, it is not a direct symptom of low blood sugar.

12. Correct answer:

A. Elevated blood glucose levels. Prednisone is a corticosteroid medication that can cause elevated blood glucose levels, especially in patients with diabetes. Corticosteroids can induce hyperglycemia by increasing insulin resistance and promoting gluconeogenesis. When a patient with diabetes is prescribed prednisone, the nurse should carefully monitor their blood glucose levels and adjust their diabetes management plan as needed, in collaboration with the healthcare provider.

Incorrect answer options:

B. Enhanced creatinine clearance. Prednisone does not typically have a direct effect on creatinine clearance. Creatinine clearance is an indicator of kidney function, and while diabetes can cause kidney damage over time, prednisone is not directly responsible for changes in creatinine clearance.

C. Increased platelet clumping. Prednisone does not directly cause increased platelet clumping. Platelet clumping is associated with an increased risk of blood clot formation, but this is not a common side effect of prednisone.

D. Increased insulin sensitivity. Prednisone does not increase insulin sensitivity; in fact, it can lead to increased insulin resistance, causing elevated blood glucose levels. This is why it is essential for the nurse to monitor blood glucose levels in patients with diabetes who are prescribed prednisone.

13. Correct answer:

C. At 11:00 AM and 5:00 PM. A diabetic patient who receives a mixture of regular and NPH insulin at 7:00 AM should be particularly vigilant for symptoms of hypoglycemia at two time intervals: 11:00 AM and 5:00 PM. Regular insulin has an onset of action within 30 minutes to 1 hour, peaks between 2 to 4 hours, and has a duration of 5 to 8 hours. NPH insulin has an onset of action within 1 to 2 hours, peaks between 4 to 12 hours, and has a duration of 18 to 24 hours. As a result, the patient is at an increased risk of hypoglycemia when the regular insulin peaks (around 11:00 AM) and when the NPH insulin peaks (around 5:00 PM).

Incorrect answer options:

A. Between 12:00 PM and 1:00 PM. This time interval is not specific enough to pinpoint the peak action times of both regular and NPH insulin.

B. At 10:00 AM and 10:00 PM. The time intervals mentioned here do not align with the peak action times of both regular and NPH insulin, which are typically around 11:00 AM and 5:00 PM, respectively.

D. Between 2:00 PM and 4:00 PM. This time interval may cover part of the peak action time for NPH insulin but does not account for the peak action time of regular insulin, which is typically around 11:00 AM.

14. Correct answer:

A. Ought to be consumed alongside meals. Metformin (Glucophage) is an oral antidiabetic medication that helps control blood sugar levels in patients with type 2 diabetes. The nurse should educate the client to take metformin alongside meals. This helps reduce gastrointestinal side effects, such as nausea and diarrhea, and allows for better absorption of the medication.

Incorrect answer options:

B. May result in weight gain. Metformin does not typically cause weight gain. In fact, it can lead to weight loss in some patients due to its ability to reduce appetite and improve insulin sensitivity.

C. Has the potential to enhance aspirin’s effects. Metformin does not have a known interaction with aspirin that would enhance its effects. However, it is important for patients to inform their healthcare provider about all medications they are taking to ensure there are no potential drug interactions.

D. Is recommended for intake during nighttime. Metformin is not specifically recommended for intake during nighttime. It is typically taken with meals, which helps with absorption and minimizes gastrointestinal side effects.

15. Correct answer:

A. Fluid imbalance: risk for deficient fluid volume. The priority nursing diagnosis for a patient admitted to the hospital with a diagnosis of diabetes insipidus is fluid imbalance: risk for deficient fluid volume. Diabetes insipidus is a condition characterized by excessive thirst and the excretion of large amounts of diluted urine. This can lead to dehydration and electrolyte imbalances, making it crucial for the nurse to address and monitor the patient’s fluid balance.

Incorrect answer options:

B. Sleep pattern disturbance due to nocturia. While sleep pattern disturbance due to nocturia is an issue in diabetes insipidus, the priority nursing diagnosis should focus on managing the patient’s fluid balance to prevent dehydration and its complications.

C. Activity intolerance related to muscle weakness. Although patients with diabetes insipidus might experience fatigue and dehydration, activity intolerance related to muscle weakness is not the priority nursing diagnosis in this case. The primary concern is managing the patient’s fluid balance.

D. Risk for impaired skin integrity related to generalized edema. Diabetes insipidus does not typically cause generalized edema. Instead, patients with this condition are more likely to experience dehydration and electrolyte imbalances due to excessive urination.

16. Correct answer:

A. Significant variations in blood pressure. Following an adrenalectomy for a client diagnosed with pheochromocytoma, the nurse should meticulously evaluate this patient for significant variations in blood pressure. Pheochromocytoma is a rare tumor of the adrenal gland that can cause the release of excessive amounts of catecholamines, such as epinephrine and norepinephrine. These hormones can lead to episodes of severe hypertension. After an adrenalectomy, the removal of the tumor may cause fluctuations in blood pressure as the body adjusts to the change in hormone levels. Monitoring and managing blood pressure is crucial to prevent complications, such as stroke or heart failure.

Incorrect answer options:

B. Low potassium levels (hypokalemia). While it is essential to monitor electrolyte levels in postoperative patients, hypokalemia is not specifically associated with adrenalectomy for pheochromocytoma.

C. Excessive sodium and water intake. Excessive sodium and water intake may lead to fluid retention and hypertension, but it is not the primary concern following an adrenalectomy for pheochromocytoma. The main focus should be on monitoring blood pressure fluctuations.

D. High blood sugar (hyperglycemia). Although blood sugar levels should be monitored in all postoperative patients, high blood sugar is not the primary concern following an adrenalectomy for pheochromocytoma. The priority is to assess and manage significant variations in blood pressure.

17. Correct answer:

A. Central obesity. A nurse should expect to find central obesity in a patient who has been receiving long-term steroid therapy. Long-term use of corticosteroids, such as prednisone, can cause a variety of side effects, including changes in fat distribution. Central obesity, also known as truncal obesity, is characterized by an accumulation of fat in the abdominal area, giving the patient a rounded or “apple-shaped” appearance.

Incorrect answer options:

B. Jaundice. Jaundice, characterized by yellowing of the skin and eyes, is not typically associated with long-term steroid therapy. It is more commonly related to liver dysfunction or hemolytic anemia.

C. Flank pain. Flank pain is not typically associated with long-term steroid therapy. It can be a symptom of kidney or urinary tract issues, such as kidney stones or infection.

D. Diminished appetite. Long-term steroid therapy often increases appetite rather than diminishing it. This increased appetite, combined with changes in fat distribution, can contribute to central obesity.

18. Correct answer:

A. Serum sodium (Na) = 120 mEq/L and low serum osmolality. In a patient with Syndrome of Inappropriate Antidiuretic Hormone (SIADH), a nurse would expect to find hyponatremia (low serum sodium levels) and low serum osmolality. SIADH is a condition where the body produces excessive amounts of antidiuretic hormone (ADH), leading to water retention and dilution of electrolytes in the blood. As a result, serum sodium levels decrease, leading to hyponatremia, and serum osmolality is reduced due to the excess water in the bloodstream.

Incorrect answer options:

B. Serum potassium (K) = 5 mEq/L and low serum osmolality. In SIADH, the primary electrolyte imbalance is hyponatremia, not hyperkalemia (high potassium levels). Low serum osmolality is expected, but it is not specifically related to potassium levels.

C. Serum sodium (Na) = 150 mEq/L and low urine osmolality. In SIADH, serum sodium levels are expected to be low, not high. The urine osmolality would be high rather than low, as the excess ADH promotes water reabsorption in the kidneys, concentrating the urine.

D. Serum potassium (K) = 3 mEq/L and high serum osmolality. In SIADH, the primary electrolyte imbalance is hyponatremia, not hypokalemia (low potassium levels). High serum osmolality is not expected in SIADH; instead, low serum osmolality is observed due to water retention.

19. Correct answer:

A. Hyperthermia blanket. A nurse should question the order for a hyperthermia blanket while caring for a patient experiencing a thyroid crisis, also known as thyroid storm. Thyroid storm is a life-threatening condition characterized by a rapid and exaggerated release of thyroid hormones, causing symptoms such as fever, tachycardia, hypertension, and altered mental status. The use of a hyperthermia blanket would increase the patient’s body temperature, which is already elevated due to the thyroid crisis. Instead, the nurse should focus on cooling measures to reduce the patient’s body temperature, such as using a cooling blanket or applying ice packs.

Incorrect answer options:

B. Intravenous fluid. Intravenous fluid is appropriate for a patient experiencing a thyroid crisis, as these patients often have fluid deficits due to increased metabolism and diaphoresis. Administering IV fluids helps to correct dehydration and maintain electrolyte balance.

C. Propylthiouracil. Propylthiouracil is an antithyroid medication that inhibits the synthesis of thyroid hormones. It is appropriate for a patient experiencing a thyroid crisis, as it helps to reduce the production and release of thyroid hormones, alleviating the symptoms of the crisis.

D. Propranolol (Inderal). Propranolol (Inderal) is a beta-blocker that helps to control the symptoms of a thyroid crisis by reducing heart rate, blood pressure, and tremors. It is appropriate for a patient experiencing a thyroid crisis, as it helps to manage the cardiovascular symptoms associated with the condition.

20. ​​Correct answer:

A. Contact the physician immediately if palpitations or nervousness occur. The most important instruction for a patient prescribed daily levothyroxine (Synthroid) is to contact the physician immediately if palpitations or nervousness occur. These symptoms may indicate an excessive dosage of the medication or the onset of hyperthyroidism, which requires prompt evaluation and possible adjustment of the medication dosage.

Incorrect answer options:

B. Reduce the consumption of juices and fruits high in potassium and calcium. This instruction is not specific to levothyroxine (Synthroid) administration. There is no need to reduce the consumption of juices and fruits high in potassium and calcium when taking this medication.

C. Take the medication at bedtime to minimize nausea and gas side effects. Levothyroxine (Synthroid) should be taken on an empty stomach, preferably in the morning, at least 30 minutes to 1 hour before breakfast. Taking it at bedtime may interfere with its absorption and effectiveness.

D. Only take the medication when experiencing symptoms. Levothyroxine (Synthroid) is prescribed to replace or supplement the thyroid hormone in hypothyroidism and should be taken consistently every day, regardless of the presence or absence of symptoms. Stopping or altering the dosage without consulting a healthcare professional can lead to fluctuations in thyroid hormone levels and exacerbate symptoms.

21. Correct answer:

C. “Glaucoma may have a genetic component, and your children over 30 years old should be screened yearly.” This response is the most appropriate because it acknowledges the patient’s concern and provides accurate information. Glaucoma can have a genetic component, and individuals with a family history of the condition are at an increased risk. It is recommended that the patient’s adult children, particularly those over 30 years old, undergo yearly glaucoma screening as part of their eye exams to detect and treat the condition early if it develops.

Incorrect answer options:

A. “There’s no need to worry; glaucoma is not a hereditary condition.” This statement is not accurate, as there is a hereditary component to glaucoma. Dismissing the patient’s concern without providing accurate information may result in a lack of necessary screening for the patient’s children.

B. “Glaucoma screening should be part of an annual eye exam for everyone over the age of 50.” While glaucoma screening should be part of an annual eye exam, particularly for those over 50, the patient’s specific concern is about the hereditary risk for her children. The response should focus on the increased risk and the need for earlier screening for those with a family history of glaucoma.

D. “Glaucoma is only hereditary if it develops before age 40, so your children are unlikely to be affected.” This statement is not accurate, as glaucoma can be hereditary regardless of the age of onset. It is essential to provide accurate information about the genetic component and the importance of early screening for those with a family history of glaucoma.

22. Correct answer:

A. Evaluate medications to identify any that may cause an increase in intraocular pressure (IOP) as a side effect. Certain medications can cause an increase in intraocular pressure (IOP), potentially worsening the condition of a patient with angle-closure glaucoma. Therefore, it’s important to review the patient’s medication list and work with the healthcare provider to adjust or discontinue any medications that could contribute to increased IOP.

Incorrect answer options:

B. Assess the patient’s level of discomfort, as they may experience significant pain until the optic nerve atrophies. This statement is incorrect because although patients with acute angle-closure glaucoma can experience significant pain, it’s not appropriate to wait for the optic nerve to atrophy. Prompt medical treatment is necessary to relieve the pain and prevent further vision loss.

C. Control blood pressure to reduce the potential loss of peripheral vision. While controlling blood pressure is generally good for overall health, it’s not specifically linked to the prevention of peripheral vision loss in angle-closure glaucoma. The main concern in angle-closure glaucoma is the elevated IOP, which needs to be managed to prevent vision loss.

D. Apply pressure to the patient’s closed eyes to alleviate intraocular pressure. This statement is incorrect because applying pressure to the eyes can actually increase IOP, potentially worsening the condition. It’s not an appropriate nursing intervention for a patient with angle-closure glaucoma.

23. Correct answer:

B. Facilitate drainage from the ear. A myringotomy, also known as tympanostomy, is a surgical procedure in which a small incision is made in the eardrum (tympanic membrane) to drain fluid from the middle ear. This is often performed when a patient has a middle ear infection (otitis media) or when fluid accumulates in the middle ear and fails to drain naturally through the Eustachian tube.

Incorrect answer options:

A. Equalize pressure on the tympanic membrane. Equalizing pressure on the tympanic membrane is a secondary effect of myringotomy. While the procedure does equalize pressure, its primary goal is to drain fluid that has built up in the middle ear.

C. Cleanse the Eustachian tube. A myringotomy does not cleanse the Eustachian tube. Its primary goal is to drain fluid from the middle ear, which can help alleviate symptoms associated with Eustachian tube dysfunction, such as ear pressure or pain, but it does not directly cleanse the tube.

D. Eliminate excess earwax buildup in the ear canal. A myringotomy is not used to eliminate excess earwax. The procedure involves an incision in the eardrum to drain fluid from the middle ear, not the removal of earwax from the ear canal.

24. Correct answer:

A. Refrain from rubbing the eye for a duration of 15-20 minutes. When a patient’s eye has been anesthetized, it is crucial to avoid rubbing or touching the eye until the anesthetic has worn off. This is because the protective reflexes of the eye are temporarily impaired, making it susceptible to injury.

Incorrect answer options:

B. Use sunglasses while exposed to direct sunlight for the following 6 hours. While protecting the eyes from direct sunlight is generally good advice, it’s not specifically related to having an eye anesthetized. Depending on the type of procedure and the specific anesthetic used, the patient may be more sensitive to light, but it’s not universally applicable.

C. Perform hourly eye irrigation to avert dryness. Frequent eye irrigation is not typically recommended after eye anesthesia. Too much irrigation could potentially lead to complications, such as infection or damage to the eye. Instead, artificial tears may be recommended if dryness is a concern.

D. Immediately wash the eye with tap water for 5 minutes. Washing the eye with tap water is not recommended unless there is a need to remove a foreign substance or chemical. In fact, it could potentially increase the risk of infection.

25. Correct answer:

C. “While my eyes are infected, I will use my personal washcloth and towel.” Conjunctivitis, commonly known as pink eye, is highly contagious. Therefore, it’s important to avoid sharing personal items like washcloths and towels, as they can easily spread the infection.

Incorrect answer options:

A. “I should wear contact lenses to help my eyes heal faster.” Contact lenses should not be worn during an active conjunctivitis infection. They can further irritate the eye and potentially spread the infection.

B. “Upon waking up, I can remove the crusty debris from my eyelashes using my fingers.” Touching the eyes with hands can potentially worsen the infection and spread it to others. It is better to use a clean, damp cloth to gently remove any discharge.

D. “Gently rubbing my eye with the back of my hand is acceptable.” Rubbing the eye can irritate it further and increase the risk of spreading the infection. It is recommended to avoid touching or rubbing the infected eye.

26. Correct answer:

A. Hydrochlorothiazide (HydroDIURIL). Hydrochlorothiazide is a diuretic that can help manage Meniere’s disease by reducing fluid volume in the body. The primary hypothesis for the cause of Meniere’s disease is an excess of fluid in the inner ear, which can lead to vertigo, tinnitus (ringing in the ear), and hearing loss. By reducing fluid levels, diuretics like Hydrochlorothiazide can help alleviate these symptoms.

Incorrect answer options:

B. Nifedipine. Nifedipine is a calcium channel blocker used primarily for the treatment of high blood pressure and angina, not typically for Meniere’s disease.

C. Propranolol. Propranolol is a beta-blocker used to treat conditions such as high blood pressure, angina, migraines, and certain types of tremors, not typically for Meniere’s disease.

D. Acetaminophen. Acetaminophen is a pain reliever and a fever reducer. While it may help with symptoms of discomfort associated with Meniere’s disease, it does not treat the root cause of the condition.

27. Correct answer:

A. Atropine (Atropisol) 1-2 drops in each eye immediately. Atropine is a mydriatic medication that dilates the pupils. In acute-angle glaucoma, where the drainage angle in the eye becomes blocked and intraocular pressure increases suddenly, dilation of the pupil can worsen the angle closure and increase the intraocular pressure even further. Therefore, the nurse should question this order.

Incorrect answer options:

B. Hydrochlorothiazide (Diuril) 25 mg PO daily. Hydrochlorothiazide is a diuretic, and it doesn’t directly affect intraocular pressure or the angle of the eye.

C. Carbachol (Isopto Carbachol) eye drops; 1 drop 2 times a day. Carbachol is a cholinergic medication that constricts the pupil and increases the outflow of aqueous humor, which can lower intraocular pressure. This would be an appropriate treatment for acute angle-closure glaucoma.

D. Warfarin (Coumadin) 5 mg PO daily. Warfarin is an anticoagulant that doesn’t directly affect intraocular pressure or the angle of the eye.

28. Correct Answer:

B. Enhances sound volume without necessarily improving hearing ability. Hearing aids primarily work by amplifying sound, making it easier for the individual to hear. They do not repair any damage in the ear or improve the actual hearing ability. Rather, they help to enhance the volume of sounds to make them more discernible to the person with hearing loss.

Incorrect answer options:

A. Offers mechanical support for impaired parts of the ear. Hearing aids do not provide mechanical support to the ear. They work by amplifying sound.

C. Stimulates the inner ear’s neural network to boost sound. While some advanced hearing aids can provide some degree of stimulation to the auditory nerve, their primary function is to amplify sound, not stimulate the neural network.

D. Repairs damage to the auditory nerve. Hearing aids do not have the capacity to repair any damage to the auditory nerve or any other part of the ear. They work by amplifying sound to assist with hearing.

29. Correct Answer:

A. Help the client to either sit or lie down. Vertigo is a sensation of feeling off-balance and can lead to a loss of balance or a fall. The nurse’s priority action when a client experiences vertigo is to help them sit or lie down immediately to prevent falls and injury. Once the client is safe, the nurse can then further assess the situation.

Incorrect answer options:

B. Determine whether the sensation is vertigo or dizziness. While differentiating between vertigo and dizziness is important for diagnosis and treatment, it is not the immediate priority. Ensuring client safety is the first concern.

C. Urge the client to continue walking to see if symptoms dissipate. Continuing to walk while experiencing vertigo puts the client at risk of falls and injuries. It is safer to have the client sit or lie down until the symptoms pass.

D. Instruct the client to sit in a chair and lower their head. While this may help some people with vertigo, the immediate concern is to prevent falls and injury by helping the client to sit or lie down.

30. Correct Answer:

B. 17-year-old engaged in physical contact sports. Retinal detachment is a serious eye condition that occurs when the retina becomes separated from the back of the eye. Risk factors for retinal detachment include severe nearsightedness, previous eye surgery, a family history of retinal detachment, certain eye diseases or disorders, and physical eye injury or trauma.

A 17-year-old engaged in physical contact sports is at a higher risk due to the potential for physical eye injury or trauma that could result in retinal detachment.

Incorrect answer options:

A. 4-year-old with amblyopia. Amblyopia, also known as lazy eye, is a vision development disorder in which an eye fails to achieve normal visual acuity, even with prescription eyeglasses or contact lenses. While amblyopia is a serious condition that should be treated, it doesn’t directly increase the risk of retinal detachment.

C. 33-year-old with severe ptosis and diplopia. Ptosis is a drooping of the upper eyelid, and diplopia is double vision. While these conditions can be troublesome, they do not directly increase the risk of retinal detachment.

D. 72-year-old with nystagmus and Bell’s palsy. Nystagmus is a condition of involuntary eye movement, and Bell’s palsy is a condition that causes sudden, temporary weakness in your facial muscles. While these conditions can affect vision and facial control, they do not directly increase the risk of retinal detachment.

31. Correct Answer:

A. Initiate measures to prevent falls. A stapedectomy is a surgical procedure that involves the removal of the stapes bone in the middle ear, which is replaced with a prosthesis. This surgery is performed to improve hearing, particularly in patients with otosclerosis, a condition where the stapes bone becomes fixated, preventing sound from reaching the inner ear.

Postoperatively, patients can often experience vertigo or a sense of imbalance due to the changes in the inner ear. Therefore, initiating measures to prevent falls is a crucial aspect of the nursing care plan for these patients. These measures can include assistance with mobilization, ensuring the patient’s environment is free of clutter, and using aids such as handrails or walking aids if needed.

Incorrect answer options:

B. Take steps to avoid aspiration. While preventing aspiration is always important in postoperative care, it’s not a specific concern related to stapedectomy.

C. Provide 2-4L/min of oxygen through a nasal cannula. There’s no specific need for supplemental oxygen for patients after a stapedectomy unless the patient has an underlying condition that necessitates it.

D. Give sedatives to encourage rest and relaxation. Sedatives aren’t typically needed after a stapedectomy. It’s important to maintain a clear sensorium to monitor for postoperative complications, such as vertigo, nausea, or hearing changes.

32. Correct Answer:

D. “I’ll opt for doing laundry this afternoon rather than going to work.” After cataract surgery, it’s important for the patient to avoid strenuous activities, including heavy lifting, bending, or straining, as they can increase intraocular pressure and potentially lead to complications such as damage to the surgical site or retinal detachment. Doing laundry, which could involve these activities, is not advisable. The patient should rest and limit activities until they have been cleared to do so by their healthcare provider.

Incorrect answer options:

A. “I’ll contact you if I experience considerable pain.” This is a correct response. Postoperative pain may suggest complications, such as infection or increased intraocular pressure. It’s important that the patient contacts their healthcare provider if they experience significant pain.

B. “I’ll keep taking my Metamucil for one more week.” This is also correct. Metamucil, a fiber supplement, is used to prevent constipation. Constipation can lead to straining during bowel movements, which can increase intraocular pressure. Therefore, preventing constipation can be beneficial for a patient recovering from cataract surgery.

C. “If it’s recommended that I rest at home and limit my activities, I’ll follow that advice.” This is the correct attitude after cataract surgery. Resting and limiting activities can help the healing process and reduce the risk of complications.

33. Correct Answer:

D. Epinephrine HCL (Epirate). Epinephrine (Epirate) is a sympathomimetic drug that, when administered as eye drops, can potentially cause systemic effects such as increased heart rate and blood pressure due to its vasoconstrictive properties. While it is primarily used to decrease intraocular pressure in the treatment of glaucoma, the potential systemic effects could theoretically exacerbate elevated intracranial pressure (ICP) by increasing cerebral perfusion pressure. Therefore, caution is advised when administering this medication to a patient with elevated ICP, and their clinical status should be closely monitored.

Incorrect answer options:

A. Artificial tears are used mainly for lubrication and relief from dry eyes. They do not affect ICP or intraocular pressure and are generally safe for use in patients with elevated ICP.

B. Betaxolol (Betoptic) is a beta-blocker eye drop used primarily for reducing intraocular pressure in conditions such as glaucoma. While it can potentially decrease cerebral blood flow, which may exacerbate ICP in susceptible individuals, it is generally considered safe unless the patient has other contraindications to beta-blockers.

C. Acetazolamide (Diamox) is often used to decrease intraocular pressure and can also reduce cerebrospinal fluid production, which may help to lower ICP. Therefore, it is generally beneficial for a patient with elevated ICP.

34. Correct answer:

D. Regularity and intensity of symptoms. Understanding the regularity and intensity of symptoms is crucial for the nurse when admitting a patient with Meniere’s disease. Meniere’s disease is a disorder of the inner ear that can cause severe dizziness (vertigo), ringing in the ears (tinnitus), hearing loss, and a feeling of fullness or congestion in the ear. The intensity and frequency of these symptoms can vary significantly between individuals and even in the same individual over time. The nurse needs to gather a detailed history of the patient’s symptoms to understand the severity of the disease, which will help guide treatment strategies and management of the condition.

Incorrect answer options:

A. Immunization history. While immunization history is essential for overall health assessment, it is not specifically crucial for a patient with Meniere’s disease. Meniere’s is not caused by an infectious agent that could be prevented by a vaccine, making this information less relevant for this specific condition.

B. Preliminary hearing evaluation. While hearing loss is a symptom of Meniere’s disease, a preliminary hearing evaluation, although useful, is not the most crucial assessment upon admission. The primary concern would be the patient’s current state, particularly the severity and frequency of vertigo, as this could lead to falls and injury.

C. Dietary history. Dietary history could be relevant, as certain dietary changes might help manage Meniere’s disease. For example, reducing salt intake can help control symptoms in some individuals. However, understanding the regularity and intensity of symptoms is still the most crucial initial assessment.

35. Correct answer:

A. Dietary habits. In this case, the initial priority when obtaining health history would be the patient’s dietary habits. The symptoms of dizziness, weakness, and palpitations may suggest nutritional deficiencies, dehydration, or imbalances in electrolytes, which can be directly linked to dietary habits. For instance, insufficient intake of iron could lead to anemia, resulting in these symptoms. Similarly, inadequate fluid or electrolyte consumption can also cause these symptoms. Therefore, understanding the patient’s dietary habits can provide critical insights into possible causes of her symptoms.

Incorrect answer options:

B. Coping mechanisms and stress tolerance. While understanding coping mechanisms and stress tolerance is a valuable part of a comprehensive health assessment, it’s not the initial priority given the described symptoms. While stress can manifest in physical symptoms, it’s more common in chronic, rather than acute, conditions.

C. Activity and exercise routines. While knowing the patient’s activity and exercise routines can be important, particularly if the symptoms occurred during or after physical exertion, it’s not the first priority. Overexertion can lead to symptoms such as dizziness and weakness, but unless these symptoms are clearly tied to physical activity, other causes should be considered first.

D. Family health status. Family health status can provide valuable information about potential genetic or hereditary conditions. However, given the acute nature of the symptoms (dizziness, weakness, palpitations), the immediate concern would be factors that could cause these symptoms in the short term, like dietary habits, rather than potential long-term genetic factors.

36. Correct answer:

D. Refrain from providing uncooked or unpeeled fresh vegetables. Patients undergoing chemotherapy, such as a child with leukemia, are at an increased risk of infection due to the treatment’s impact on the immune system. Raw and unpeeled vegetables can harbor bacteria and other harmful microorganisms that can cause infection. To reduce this risk, it is advised to provide the child with cooked vegetables and to peel fruits and vegetables that will be consumed raw.

Incorrect answer options:

A. Administer aspirin for pain or fever. Aspirin should be avoided in children due to the risk of Reye’s syndrome, a rare but serious condition that can cause swelling in the liver and brain. Instead, acetaminophen is often recommended for pain and fever in children.

B. Offer a diet with low protein and high carbohydrates. Children undergoing chemotherapy often require more protein to support healing and the immune system. A balanced diet, with appropriate amounts of protein, carbohydrates, and healthy fats, is generally recommended.

C. Encourage the use of humidifiers throughout the home. While humidifiers can help soothe certain side effects of chemotherapy, such as dry mouth or throat, they can also harbor mold and bacteria if not properly maintained. Therefore, they should be used with caution and cleaned regularly.

37. Correct answer:

A. A toddler primarily consumes milk as a source of nutrition. Toddlers who consume a large amount of milk and lack a balanced diet can be at risk for iron deficiency anemia. Milk is low in iron and can also decrease the absorption of iron from other foods, leading to a deficiency. This condition is sometimes referred to as “milk anemia.” Additionally, a milk-heavy diet can displace other foods rich in iron, further exacerbating the risk.

Incorrect answer options:

B. A cancer patient undergoing radiation therapy twice weekly. While cancer patients undergoing radiation therapy can develop anemia, it is typically not due to iron deficiency but rather due to the impact of the therapy on the bone marrow’s ability to produce red blood cells.

C. A 15-year-old patient experiencing a sickle cell crisis. People with sickle cell disease can experience anemia, but this is not due to iron deficiency. It is caused by the rapid destruction of sickled cells, which have a shorter lifespan than normal red blood cells.

D. A patient with chronic respiratory disease receiving long-term corticosteroid treatment. While long-term corticosteroid use can have many side effects, it is not typically associated with iron deficiency anemia. These medications can cause changes in red blood cell production, but they do not directly affect iron levels.

38. Correct answer:

B. Begin the transfusion within 30 minutes of obtaining the blood. When initiating a blood transfusion, it is important to begin the transfusion within 30 minutes of obtaining the blood from the blood bank. This is because blood products can begin to deteriorate or become contaminated if left at room temperature for too long.

Incorrect answer options:

A. Observe the client’s vital signs during the initial 5 minutes. While observing the client’s vital signs is important during a blood transfusion, it is generally recommended to monitor vital signs before, 15 minutes after initiation, and at the end of the transfusion. This can help detect any potential adverse reactions to the transfusion.

C. Start an IV with 5% dextrose in water (D5W) to ensure a patent access site. D5W is not used with blood transfusions because it can cause the blood cells to hemolyze or rupture. Normal saline (0.9% sodium chloride) is the only solution compatible with blood products.

D. Infuse the blood quickly to avoid clot formation. Blood should not be infused too quickly, especially at the start of the transfusion. This could overwhelm the patient’s circulatory system and lead to complications. The blood should be infused slowly at first (generally over the first 15 minutes), then the rate can be increased if there are no signs of a reaction.

39. Correct answer:

A. Flush the eye thoroughly with water or saline solution. If a client accidentally splashes an unknown chemical into their eyes, the first and most crucial action is to flush the eyes thoroughly with water or a saline solution. This helps to remove the chemical from the eye and can potentially limit the damage caused by the chemical. The client should continue flushing the eye for at least 15-20 minutes, or until they can reach professional medical help.

Incorrect answer options:

B. Place a pad soaked in sterile saline solution over the eye. While a saline-soaked pad may offer some relief, it’s not sufficient for an immediate response to a chemical exposure. It won’t effectively remove the chemical from the eye, which is necessary to minimize damage.

C. Visit the nearest emergency room. While visiting an emergency room is crucial after a chemical eye injury, it’s not the immediate action. Before leaving for the emergency room, the client should first flush their eye to remove as much of the chemical as possible.

D. Rub the affected eye vigorously to eliminate debris or chemicals. Rubbing the eye could potentially cause more harm by exacerbating the damage caused by the chemical. The best initial action is to flush the eye with water or saline.

40. Correct answer:

B. Halt the transfusion, detach the blood tubing, and commence a primary infusion of normal saline solution. The symptoms the client is presenting (lower back pain, headache, restlessness) are suggestive of a possible transfusion reaction, which can be a serious and potentially life-threatening event. The immediate course of action should be to stop the transfusion and remove the blood tubing. This is done to stop the introduction of more potentially reactive blood products into the client’s system. The nurse should then start a primary infusion of normal saline solution to maintain IV access and provide a way to administer medications if needed.

Incorrect answer options:

A. Decelerate the infusion, assess vital signs, and review the client’s history of transfusion reactions. While these actions are important, they should not be the first step in response to the client’s symptoms. The priority is to stop the transfusion immediately to prevent further potential harm.

C. Double-check the blood unit for accurate identification numbers and cross-match details. While ensuring the correct identification numbers and cross-match details is important before starting a transfusion, if a reaction is occurring, the immediate action should be to stop the transfusion, not to check these details.

D. Give the client a dose of diuretic medication. Administering diuretics isn’t the initial response to a suspected transfusion reaction. The priority is stopping the transfusion and maintaining the client’s stability.

41. Correct answer:

A. 0.9% NaCl. Normal saline (0.9% sodium chloride) is the preferred solution for intravenous infusion before and during blood transfusion. This isotonic solution is compatible with blood products, and it doesn’t cause red blood cells to hemolyze (burst).

Incorrect answer options:

B. 10% dextrose in water (D10W). Dextrose solutions, including D10W, can cause hemolysis of red blood cells. Therefore, they are not suitable to be used with blood transfusions.

C. Lactated Ringer’s solution. Lactated Ringer’s solution, though often used for fluid resuscitation, is not compatible with blood products and can cause clotting. Therefore, it should not be used as the primary fluid during a blood transfusion.

D. D5W/0.45% NaCl. D5W/0.45% NaCl, a hypotonic solution, can cause hemolysis of red blood cells and should not be used with blood transfusions.

42. Correct answer:

A. Fresh Frozen Plasma (FFP). In the case of disseminated intravascular coagulation (DIC), where there is abnormal activation of the clotting cascade leading to the formation of small blood clots throughout the body and eventual bleeding due to depletion of clotting factors, Fresh Frozen Plasma (FFP) is often used. FFP contains all coagulation factors and is beneficial for replacing these factors in patients with DIC.

Incorrect answer options:

B. Whole blood. Whole blood transfusions are rarely used nowadays and are typically reserved for massive hemorrhage or when a patient has lost 25% or more of their blood volume. In the case of DIC, where there is a deficiency in clotting factors, FFP is more appropriate.

C. Packed Red Blood Cells. Although packed red blood cells (PRBCs) might be used if the patient is experiencing significant bleeding and anemia, PRBCs do not contain the necessary clotting factors that a DIC patient needs. FFP would be more beneficial in this case.

D. Potassium chloride. Potassium chloride is used to treat or prevent potassium deficiency (hypokalemia). It’s not typically used in the treatment of DIC as it doesn’t address the underlying issue of clotting factor depletion.

43. Correct answer:

A. Headaches, dyspnea, and claudication. Polycythemia vera is a rare, chronic condition characterized by the overproduction of red blood cells in the bone marrow, which causes the blood to thicken. Common symptoms include headaches due to increased blood viscosity and consequent reduced cerebral blood flow, dyspnea, and claudication due to impaired oxygen delivery to the muscles, especially during exercise.

Incorrect answer options:

B. Elevated fatigue and bleeding tendencies. Although fatigue can be a symptom of polycythemia vera, it’s not typically associated with bleeding tendencies. On the contrary, patients with this condition are often at a higher risk for blood clot formation due to increased blood viscosity.

C. Hemoglobin levels below 13 mg/dl. In polycythemia vera, the body overproduces red blood cells, leading to an increase in hemoglobin levels, not a decrease. Therefore, we’d expect hemoglobin levels to be higher than normal, not below the standard reference range.

D. Low platelet count and frequent infections. Polycythemia vera typically involves an increase in red blood cells, and sometimes also white blood cells and platelets, so a low platelet count would be unusual. Moreover, while an increase in white blood cells might suggest an increased risk of infection, it doesn’t necessarily translate into frequent infections in the context of polycythemia vera.

44. Correct answer:

A. Monthly vitamin B12 injections will be required. Pernicious anemia is a type of anemia that occurs due to a deficiency of vitamin B12, often caused by impaired absorption resulting from the lack of intrinsic factor, a protein produced by the stomach. Treatment for pernicious anemia usually involves regular vitamin B12 injections to bypass the absorption issue. Monthly injections are a common treatment plan, but the frequency may vary depending on the patient’s specific needs and healthcare provider recommendations.

Incorrect answer options:

B. Reduce consumption of leafy green vegetables due to elevated vitamin K levels. This recommendation is not related to pernicious anemia. Leafy green vegetables are rich in vitamin K, which is essential for blood clotting. However, pernicious anemia is related to vitamin B12 deficiency, not vitamin K.

C. Consume daily iron supplements to enhance red blood cell production. Iron supplements are typically prescribed for patients with iron-deficiency anemia, not pernicious anemia. In pernicious anemia, the primary issue is the lack of vitamin B12, which affects red blood cell production and maturation.

D. Coagulation studies are essential for evaluating medications. Coagulation studies are not typically required for monitoring pernicious anemia or its treatment. These studies assess blood clotting function and would be more relevant for patients with clotting disorders or those taking anticoagulant medications.

45. Correct answer:

A. Hydration, oxygenation, pain management. During a sickle cell crisis, a patient’s sickle-shaped red blood cells can stick together and block blood flow, causing pain and potential damage to tissues and organs. The priorities of care for these patients typically include hydration, which can help to thin the blood and prevent further sickling; oxygenation, which can help to reduce sickling and improve tissue oxygenation; and pain management, as sickle cell crises can be extremely painful.

Incorrect answer options:

B. Nutrition, hydration, electrolyte balance. While maintaining adequate nutrition and electrolyte balance are important aspects of overall patient care, they are not the primary focus during a sickle cell crisis. The immediate concerns are usually hydration, oxygenation, and pain management.

C. Hydration, pain management, electrolyte balance. Again, while maintaining electrolyte balance is an important aspect of care, oxygenation is a higher priority during a sickle cell crisis due to the risk of tissue hypoxia from blocked blood vessels.

D. Hydration, oxygenation, electrolyte balance. While hydration and oxygenation are key priorities, this option omits pain management, which is a crucial aspect of care during a sickle cell crisis.

46. Correct answer:

C. The ability of the client to speak. The cuff of an endotracheal tube (ET) is inflated to create a seal between the tube and the trachea. This prevents air from leaking around the tube during mechanical ventilation and also helps prevent aspiration. If a patient can speak, it suggests that air is passing around the ET tube and the vocal cords, which should not occur if the cuff is properly inflated. Therefore, the ability to speak indicates a potential cuff leak.

Incorrect answer options:

A. Increased crackles (rales) in the left lung field. Crackles are a sign of fluid in the lungs, such as in the case of pneumonia or heart failure. They are not typically indicative of a leak in an ET tube cuff.

B. Decreased pulse rate. A decreased pulse rate is not typically associated with a leak in an ET tube cuff. Rather, it could indicate a variety of conditions such as a response to medications, heart disease, or other physiological changes.

D. Wheezing detected upon auscultation. Wheezing indicates narrowed airways and is often associated with conditions such as asthma or chronic obstructive pulmonary disease (COPD). It is not typically a sign of a leak in an ET tube cuff.

47. Correct answer:

C. PCO2 levels rise from 35 to 45 mm Hg. An increase in partial pressure of carbon dioxide (PCO2) levels in the blood is a common sign of hypoventilation or ineffective airway clearance. This could be due to retained mucus blocking the airways and reducing gas exchange, leading to a buildup of carbon dioxide in the blood. Normal PCO2 levels range from about 35 to 45 mm Hg, so an increase within this range, especially in the context of respiratory symptoms, could suggest that the client is not effectively clearing their secretions.

Incorrect answer options:

A. Chest X-ray reveals right-sided pleural fluid. Pleural fluid could indicate a condition like pleural effusion, but it does not directly indicate ineffective clearance of mucus secretions.

B. Decreased heart rate. A decreased heart rate is not directly associated with ineffective clearance of mucus secretions. It could indicate a variety of conditions or responses to medications, but it is not a typical sign of poor airway clearance.

D. A few scattered crackles in RLL detected during auscultation. While crackles can sometimes indicate the presence of fluid in the lungs, a few scattered crackles in the RLL after a lobectomy does not definitively indicate ineffective clearance of mucus secretions. It could simply be a normal postoperative finding or could be related to a small amount of fluid in the lungs.

48. Correct answer:

C. The client will need to limit activity at home due to the requirement for oxygen. Clients with COPD who are on oxygen therapy may need to adjust their level of physical activity, depending on their individual tolerance and the severity of their symptoms. It is crucial to balance activity with rest periods to avoid excessive fatigue and worsening of symptoms. However, it does not mean that they have to completely limit their activities. Regular, moderate exercise is beneficial for maintaining muscle strength, enhancing overall well-being, and improving respiratory function.

Incorrect answer options:

A. The client can discontinue oxygen usage once he feels better. This is incorrect because oxygen therapy for COPD is usually a long-term treatment. It should not be discontinued without medical advice, even if the client feels better.

B. Oxygen use will completely alleviate the client’s shortness of breath. While oxygen can help alleviate shortness of breath, it does not completely eliminate this symptom. The degree of relief can vary depending on the severity of the disease and the client’s individual response to therapy.

D. Oxygen therapy can cure COPD. Oxygen therapy does not cure COPD. It is a chronic, progressive disease. Oxygen therapy can help manage symptoms and improve quality of life, but it does not reverse the underlying lung damage caused by COPD.

49. Correct answer:

C. The catheter is introduced into the endotracheal tube until resistance is encountered, with intermittent suction applied during withdrawal. This is the proper technique for endotracheal suctioning. The catheter is inserted through the endotracheal tube until resistance is felt (which indicates the catheter has reached the carina), and then it is withdrawn slightly before applying intermittent suction during withdrawal. This method helps to clear the airway of secretions while reducing the risk of damage to the airway and lung tissue.

Incorrect answer options:

A. The catheter is placed within the endotracheal tube and remains there for ongoing suctioning. Leaving a suction catheter in place can cause trauma to the tracheal lining and increase the risk of infection.

B. The catheter is initially inserted through the nose to suction the upper airway, then removed and inserted into the endotracheal tube to suction the lower airway. This is not a standard practice. The same catheter should not be used for suctioning both the upper and lower airways due to the risk of cross-contamination and infection.

D. Suctioning via an endotracheal tube is not required. Suctioning via an endotracheal tube is often necessary to maintain a patent airway, prevent aspiration, and improve oxygenation and ventilation.

50. Correct Answer:

B. “Organize a routine where your husband completes essential tasks before noon, allowing him to rest in the afternoon and evening.” This advice can help the spouse manage the patient’s care at home. For a person with COPD, energy conservation is crucial. By completing essential tasks in the morning when energy levels are typically higher, the patient can rest later in the day when fatigue often sets in. This strategy can help manage symptoms of breathlessness and prevent overexertion.

Incorrect answer options:

A. “Your husband may experience shortness of breath during activities, but this won’t hurt him.” This statement, while potentially intended to be reassuring, may be misleading. Shortness of breath in a patient with COPD can be distressing and may signify that activities are being performed at a pace or intensity that is too strenuous. It’s important to encourage the spouse to understand that while some shortness of breath may be expected, monitoring the severity and duration is crucial.

C. “It’s best to avoid emotionally charged situations that can exacerbate his shortness of breath.” While it is true that stress and emotional upheaval can exacerbate symptoms of COPD, including shortness of breath, it’s not practical or healthy to avoid all emotional situations. Instead, it would be better to provide information on strategies to manage stress, such as relaxation techniques or seeking support from mental health professionals.

D. “Your husband’s shortness of breath will inevitably worsen over time, and there is no way to prevent it.” This statement is both overly pessimistic and inaccurate. While COPD is a progressive disease, the rate at which it progresses can vary greatly between individuals. Additionally, with appropriate management including medication, exercise, and lifestyle modifications, symptoms can be managed and the progression of the disease can potentially be slowed.